It is primarily by raising interest rates that central bankers curb inflation, but an increase in interest rates take...

filozinni on August 17, 2020

Why is C correct and E wrong?

Could anyone please explain what makes C correct over E? Thank you very much!

Replies
Create a free account to read and take part in forum discussions.

Already have an account? log in

shunhe on August 19, 2020

Hi @filozinni,

Thanks for the question! So this is a pretty complicated argument, so let’s walk through it. Central bankers curb inflation primarily by raising interest rates, but that takes up to two years to work. So they try to raise interest rates before inflation becomes excessive, before inflation isn’t readily apparent. But unless it is readily apparent, then the interest rates hikes seem unnecessary. So their success in temporarily restraining inflation might make it harder for them to ward off future inflation without incurring the public’s wrath.

So what’s the main conclusion here? It seems like it’s the last sentence, and the word “so” does support that. And everything else works towards supporting this. Everything else, then, is a premise (or possibly intermediary conclusion) that supports it. And I think we can say this is the general structure:

P: Central bankers curb inflation primarily by raising interest rates
P: This can take up to two years to work, so they raise interest rates before the inflation’s readily apparent
P: Unless inflation is readily apparent, the interest rates hikes seem unnecessary.
C: Central bankers’ success in temporarily restraining inflation might make it harder for them to ward off future inflation without incurring the public’s wrath.

So now we’re asked for the role played that central bankers curb inflation primarily by raising interest rates. OK, well, we know that this is going to be a premise. It supports the conclusion, but isn’t supported by anything else. So we just need to find language that describes a premise. And that’s described in (C), a premise that’s offered in support of the conclusion (and the description of the conclusion matches up with what we said it was).

(E), on the other hand, says that it’s a premise offered in support of another conclusion: that unless inflation is readily apparent, interest rate hikes generally will be perceived as needlessly restraining a growing economy. But is that a conclusion? No, it’s not, it’s a premise! It’s not supported by anything else. So since that statement isn’t a conclusion, (E) can’t be the correct answer.

Hope this helps! Feel free to ask any other questions that you might have.

filozinni on August 19, 2020

This helps a lot, thanks for the explanation @Shunhe!